Fix subscripting (r_12 -> r_{12}) in Serway and Jewett v8's 25.21.
authorW. Trevor King <wking@tremily.us>
Wed, 25 Jul 2012 12:40:44 +0000 (08:40 -0400)
committerW. Trevor King <wking@tremily.us>
Wed, 25 Jul 2012 12:40:44 +0000 (08:40 -0400)
latex/problems/Serway_and_Jewett_8/problem25.21.tex

index 116769bb2850cc55467c4a06644a3e3645333153..641ddd4f866f53310d12549ecdb87116ce3f438a 100644 (file)
@@ -11,7 +11,7 @@ on the $y$ axis at $y=0.500\U{m}$.
 \begin{solution}
 \Part{a}
 \begin{equation}
-  V_3 = k_e \frac{q_1}{r_13} + k_e \frac{q_2}{r_12}
+  V_3 = k_e \frac{q_1}{r_{13}} + k_e \frac{q_2}{r_{12}}
     = 2 k_e \frac{2\U{$\mu$C}}{\sqrt{(1.00\U{m})^2 + (0.500\U{m})^2}}
     = \ans{32.2\U{kV}}
 \end{equation}